Continuous function:











up vote
0
down vote

favorite













Prove: If $f$ is continuous on $I=(0,1]$ and $lim_{x to 0^+}f(x)=L$ is finite then $f$ is bounded on $I$




I'm not too sure how to start this problem, I know $f(x)$ doesn't attain its least upper bound of $0$.



And because $f(x)$ is continuous on $I=(0,1]$ then it is bounded everywhere in this interval but not at $0$.










share|cite|improve this question




























    up vote
    0
    down vote

    favorite













    Prove: If $f$ is continuous on $I=(0,1]$ and $lim_{x to 0^+}f(x)=L$ is finite then $f$ is bounded on $I$




    I'm not too sure how to start this problem, I know $f(x)$ doesn't attain its least upper bound of $0$.



    And because $f(x)$ is continuous on $I=(0,1]$ then it is bounded everywhere in this interval but not at $0$.










    share|cite|improve this question


























      up vote
      0
      down vote

      favorite









      up vote
      0
      down vote

      favorite












      Prove: If $f$ is continuous on $I=(0,1]$ and $lim_{x to 0^+}f(x)=L$ is finite then $f$ is bounded on $I$




      I'm not too sure how to start this problem, I know $f(x)$ doesn't attain its least upper bound of $0$.



      And because $f(x)$ is continuous on $I=(0,1]$ then it is bounded everywhere in this interval but not at $0$.










      share|cite|improve this question
















      Prove: If $f$ is continuous on $I=(0,1]$ and $lim_{x to 0^+}f(x)=L$ is finite then $f$ is bounded on $I$




      I'm not too sure how to start this problem, I know $f(x)$ doesn't attain its least upper bound of $0$.



      And because $f(x)$ is continuous on $I=(0,1]$ then it is bounded everywhere in this interval but not at $0$.







      real-analysis analysis






      share|cite|improve this question















      share|cite|improve this question













      share|cite|improve this question




      share|cite|improve this question








      edited yesterday









      idea

      2,07121023




      2,07121023










      asked yesterday









      Guy

      133




      133






















          3 Answers
          3






          active

          oldest

          votes

















          up vote
          3
          down vote













          A quick way to show this is by extending f to $[0,1]$: define $f(0)=L$ and note that f is now continuous on a compact set. We know that continuous functions on compacts are bounded, whence the result.






          share|cite|improve this answer




























            up vote
            0
            down vote













            HINT: Your last sentence correctly identifies why $f$ is bounded everywhere except near $0$. Near $0$, $f$ is bounded for a different reason: the values of $f$ are near $L$. So split your interval into $(0,delta)$ and $[delta, 1]$, then prove $f$ is bounded on each of these intervals for different reasons.






            share|cite|improve this answer




























              up vote
              0
              down vote













              Suppose that $f$ is not bounded. Then, for each $ninmathbb N$, there is a $a_nin(0,1]$ such that $bigllvert f(a_n)bigrrvertgeqslant n$. The sequence $(a_n)_{ninmathbb N}$ is bounded and therefore it has a convergent subsequence. Without loss of generality, we can assume that $(a_n)_{ninmathbb N}$ converges. Let $l$ be its limit. Then $lin[0,1]$ and there are two possibilities:





              1. $lin(0,1]$. Then, since $f$ is continuous at $l$, $lim_{ntoinfty}f(a_n)=f(l)$, which is impossible, since $bigl(f(a_n)bigr)_{ninmathbb N}$ is unbounded.


              2. $l=0$. Then $lim_{ntoinfty}f(a_n)=L$. Again, this is impossible, since $bigl(f(a_n)bigr)_{ninmathbb N}$ is unbounded.






              share|cite|improve this answer





















                Your Answer





                StackExchange.ifUsing("editor", function () {
                return StackExchange.using("mathjaxEditing", function () {
                StackExchange.MarkdownEditor.creationCallbacks.add(function (editor, postfix) {
                StackExchange.mathjaxEditing.prepareWmdForMathJax(editor, postfix, [["$", "$"], ["\\(","\\)"]]);
                });
                });
                }, "mathjax-editing");

                StackExchange.ready(function() {
                var channelOptions = {
                tags: "".split(" "),
                id: "69"
                };
                initTagRenderer("".split(" "), "".split(" "), channelOptions);

                StackExchange.using("externalEditor", function() {
                // Have to fire editor after snippets, if snippets enabled
                if (StackExchange.settings.snippets.snippetsEnabled) {
                StackExchange.using("snippets", function() {
                createEditor();
                });
                }
                else {
                createEditor();
                }
                });

                function createEditor() {
                StackExchange.prepareEditor({
                heartbeatType: 'answer',
                convertImagesToLinks: true,
                noModals: true,
                showLowRepImageUploadWarning: true,
                reputationToPostImages: 10,
                bindNavPrevention: true,
                postfix: "",
                imageUploader: {
                brandingHtml: "Powered by u003ca class="icon-imgur-white" href="https://imgur.com/"u003eu003c/au003e",
                contentPolicyHtml: "User contributions licensed under u003ca href="https://creativecommons.org/licenses/by-sa/3.0/"u003ecc by-sa 3.0 with attribution requiredu003c/au003e u003ca href="https://stackoverflow.com/legal/content-policy"u003e(content policy)u003c/au003e",
                allowUrls: true
                },
                noCode: true, onDemand: true,
                discardSelector: ".discard-answer"
                ,immediatelyShowMarkdownHelp:true
                });


                }
                });














                 

                draft saved


                draft discarded


















                StackExchange.ready(
                function () {
                StackExchange.openid.initPostLogin('.new-post-login', 'https%3a%2f%2fmath.stackexchange.com%2fquestions%2f3004988%2fcontinuous-function%23new-answer', 'question_page');
                }
                );

                Post as a guest















                Required, but never shown

























                3 Answers
                3






                active

                oldest

                votes








                3 Answers
                3






                active

                oldest

                votes









                active

                oldest

                votes






                active

                oldest

                votes








                up vote
                3
                down vote













                A quick way to show this is by extending f to $[0,1]$: define $f(0)=L$ and note that f is now continuous on a compact set. We know that continuous functions on compacts are bounded, whence the result.






                share|cite|improve this answer

























                  up vote
                  3
                  down vote













                  A quick way to show this is by extending f to $[0,1]$: define $f(0)=L$ and note that f is now continuous on a compact set. We know that continuous functions on compacts are bounded, whence the result.






                  share|cite|improve this answer























                    up vote
                    3
                    down vote










                    up vote
                    3
                    down vote









                    A quick way to show this is by extending f to $[0,1]$: define $f(0)=L$ and note that f is now continuous on a compact set. We know that continuous functions on compacts are bounded, whence the result.






                    share|cite|improve this answer












                    A quick way to show this is by extending f to $[0,1]$: define $f(0)=L$ and note that f is now continuous on a compact set. We know that continuous functions on compacts are bounded, whence the result.







                    share|cite|improve this answer












                    share|cite|improve this answer



                    share|cite|improve this answer










                    answered yesterday









                    Sorin Tirc

                    5039




                    5039






















                        up vote
                        0
                        down vote













                        HINT: Your last sentence correctly identifies why $f$ is bounded everywhere except near $0$. Near $0$, $f$ is bounded for a different reason: the values of $f$ are near $L$. So split your interval into $(0,delta)$ and $[delta, 1]$, then prove $f$ is bounded on each of these intervals for different reasons.






                        share|cite|improve this answer

























                          up vote
                          0
                          down vote













                          HINT: Your last sentence correctly identifies why $f$ is bounded everywhere except near $0$. Near $0$, $f$ is bounded for a different reason: the values of $f$ are near $L$. So split your interval into $(0,delta)$ and $[delta, 1]$, then prove $f$ is bounded on each of these intervals for different reasons.






                          share|cite|improve this answer























                            up vote
                            0
                            down vote










                            up vote
                            0
                            down vote









                            HINT: Your last sentence correctly identifies why $f$ is bounded everywhere except near $0$. Near $0$, $f$ is bounded for a different reason: the values of $f$ are near $L$. So split your interval into $(0,delta)$ and $[delta, 1]$, then prove $f$ is bounded on each of these intervals for different reasons.






                            share|cite|improve this answer












                            HINT: Your last sentence correctly identifies why $f$ is bounded everywhere except near $0$. Near $0$, $f$ is bounded for a different reason: the values of $f$ are near $L$. So split your interval into $(0,delta)$ and $[delta, 1]$, then prove $f$ is bounded on each of these intervals for different reasons.







                            share|cite|improve this answer












                            share|cite|improve this answer



                            share|cite|improve this answer










                            answered yesterday









                            Y. Forman

                            11.3k423




                            11.3k423






















                                up vote
                                0
                                down vote













                                Suppose that $f$ is not bounded. Then, for each $ninmathbb N$, there is a $a_nin(0,1]$ such that $bigllvert f(a_n)bigrrvertgeqslant n$. The sequence $(a_n)_{ninmathbb N}$ is bounded and therefore it has a convergent subsequence. Without loss of generality, we can assume that $(a_n)_{ninmathbb N}$ converges. Let $l$ be its limit. Then $lin[0,1]$ and there are two possibilities:





                                1. $lin(0,1]$. Then, since $f$ is continuous at $l$, $lim_{ntoinfty}f(a_n)=f(l)$, which is impossible, since $bigl(f(a_n)bigr)_{ninmathbb N}$ is unbounded.


                                2. $l=0$. Then $lim_{ntoinfty}f(a_n)=L$. Again, this is impossible, since $bigl(f(a_n)bigr)_{ninmathbb N}$ is unbounded.






                                share|cite|improve this answer

























                                  up vote
                                  0
                                  down vote













                                  Suppose that $f$ is not bounded. Then, for each $ninmathbb N$, there is a $a_nin(0,1]$ such that $bigllvert f(a_n)bigrrvertgeqslant n$. The sequence $(a_n)_{ninmathbb N}$ is bounded and therefore it has a convergent subsequence. Without loss of generality, we can assume that $(a_n)_{ninmathbb N}$ converges. Let $l$ be its limit. Then $lin[0,1]$ and there are two possibilities:





                                  1. $lin(0,1]$. Then, since $f$ is continuous at $l$, $lim_{ntoinfty}f(a_n)=f(l)$, which is impossible, since $bigl(f(a_n)bigr)_{ninmathbb N}$ is unbounded.


                                  2. $l=0$. Then $lim_{ntoinfty}f(a_n)=L$. Again, this is impossible, since $bigl(f(a_n)bigr)_{ninmathbb N}$ is unbounded.






                                  share|cite|improve this answer























                                    up vote
                                    0
                                    down vote










                                    up vote
                                    0
                                    down vote









                                    Suppose that $f$ is not bounded. Then, for each $ninmathbb N$, there is a $a_nin(0,1]$ such that $bigllvert f(a_n)bigrrvertgeqslant n$. The sequence $(a_n)_{ninmathbb N}$ is bounded and therefore it has a convergent subsequence. Without loss of generality, we can assume that $(a_n)_{ninmathbb N}$ converges. Let $l$ be its limit. Then $lin[0,1]$ and there are two possibilities:





                                    1. $lin(0,1]$. Then, since $f$ is continuous at $l$, $lim_{ntoinfty}f(a_n)=f(l)$, which is impossible, since $bigl(f(a_n)bigr)_{ninmathbb N}$ is unbounded.


                                    2. $l=0$. Then $lim_{ntoinfty}f(a_n)=L$. Again, this is impossible, since $bigl(f(a_n)bigr)_{ninmathbb N}$ is unbounded.






                                    share|cite|improve this answer












                                    Suppose that $f$ is not bounded. Then, for each $ninmathbb N$, there is a $a_nin(0,1]$ such that $bigllvert f(a_n)bigrrvertgeqslant n$. The sequence $(a_n)_{ninmathbb N}$ is bounded and therefore it has a convergent subsequence. Without loss of generality, we can assume that $(a_n)_{ninmathbb N}$ converges. Let $l$ be its limit. Then $lin[0,1]$ and there are two possibilities:





                                    1. $lin(0,1]$. Then, since $f$ is continuous at $l$, $lim_{ntoinfty}f(a_n)=f(l)$, which is impossible, since $bigl(f(a_n)bigr)_{ninmathbb N}$ is unbounded.


                                    2. $l=0$. Then $lim_{ntoinfty}f(a_n)=L$. Again, this is impossible, since $bigl(f(a_n)bigr)_{ninmathbb N}$ is unbounded.







                                    share|cite|improve this answer












                                    share|cite|improve this answer



                                    share|cite|improve this answer










                                    answered yesterday









                                    José Carlos Santos

                                    139k18111203




                                    139k18111203






























                                         

                                        draft saved


                                        draft discarded



















































                                         


                                        draft saved


                                        draft discarded














                                        StackExchange.ready(
                                        function () {
                                        StackExchange.openid.initPostLogin('.new-post-login', 'https%3a%2f%2fmath.stackexchange.com%2fquestions%2f3004988%2fcontinuous-function%23new-answer', 'question_page');
                                        }
                                        );

                                        Post as a guest















                                        Required, but never shown





















































                                        Required, but never shown














                                        Required, but never shown












                                        Required, but never shown







                                        Required, but never shown

































                                        Required, but never shown














                                        Required, but never shown












                                        Required, but never shown







                                        Required, but never shown







                                        Popular posts from this blog

                                        Can a sorcerer learn a 5th-level spell early by creating spell slots using the Font of Magic feature?

                                        Does disintegrating a polymorphed enemy still kill it after the 2018 errata?

                                        A Topological Invariant for $pi_3(U(n))$